LSAT and Law School Admissions Forum

Get expert LSAT preparation and law school admissions advice from PowerScore Test Preparation.

 Administrator
PowerScore Staff
  • PowerScore Staff
  • Posts: 8950
  • Joined: Feb 02, 2011
|
#32461
Complete Question Explanation

Weaken—CE. The correct answer choice is (A)

The author of this stimulus leads off with the conclusion, that “animal feed should not include genetically modified plants.” To support this conclusion, the author points to a study in which researchers fed one group of laboratory rats genetically modified potatoes for thirty days, while another set of rats were fed a “normal diet of foods that were not genetically modified.” The rats fed genetically modified potatoes tended to develop “intestinal deformities and a weakened immune system,” while the rats fed a normal diet did not.

As is often the case on the LSAT, there appear to be some issues with this study. Most glaringly, we do not know why potatoes were chosen as the genetically modified food, nor whether the rats were fed only potatoes, as opposed to a “normal diet of foods.” If the researchers both changed the rats’ normal diet and added genetically modified food, having both of those changes made simultaneously would cloud the issue of causation. Put another way, did the rats develop medical issues because they ate genetically modified food, or did those issues result from their thirty day diet of potatoes, regardless of the genetic modifications?

A separate issue is the generalization of the results from a study of rats to a conclusion about animal feed generally. Even if we assume that the rats’ medical issues resulted from their consumption of genetically modified potatoes, that evidence does not establish that other animals would be affected the same way the rats were affected.

The question stem indicates that this is a Weaken question. We can prephrase that the way the study was conducted, in which the researchers apparently both changed the rats’ diet and exposed the rats to genetically modified foods, makes the causation ambiguous, so that it may not be the case that genetically modified plants should be kept out of the rats’ feed, let alone animal feed.

Answer choice (A): This is the correct answer choice, because it introduces as fact our suspicion that the researchers not only introduced genetically modified food, but changed the rats’ normal diet as well. As discussed above, this information weakens the causal conclusion by raising the change from the rats’ normal diet, independent of the introduction of modified food, as an alternate cause for the negative change in the rats’ health.

Answer choice (B): Nothing in the stimulus suggests that when within the thirty day period the rats ate the majority of the modified potatoes has any impact on the causal relationship between the genetic modifications and the damage to the rats.

Answer choice (C): This answer choice has no connection to genetically modified food, and is irrelevant to the conclusion.

Answer choice (D): It is not clear what nutritional value potatoes have for rats. This answer choice may simply be saying that potatoes have zero nutritional value to rats with or without genetic modification. In any event, this comparison does not impact the implied causal relationship between genetic modification and the medical problems experienced by the rats.

Answer choice (E): This is a very tempting answer choice, because it explicitly mentions causality in a way that is negative for the researchers. Some taking the test thought that the inability of the researchers to explain how the genetic modifications would have caused the rats’ medical problems weakens the causal conclusion. However, the inability to explain a causal mechanism does not show there is not causality present.
 ray57
  • Posts: 9
  • Joined: Aug 09, 2019
|
#67702
Hello,

I put answer choice C for this one and I am not sure why it is incorrect. In your explanation you say that C is not correct because it doesn't mention genetically modified food but neither does the correct answer, A. I chose C because it undermines the conclusion by suggesting a different cause, that lab animals COULD be fed genetically modified plants since intestinal deformities are "common at birth" thereby ruling out the author's conclusion that the genetically modified food is to blame. For answer choice A, how do we know that the rats normal diet of foods didn't include potatoes? I thought C was the surefire answer here. Can you please explain why C is wrong? Thanks for your help.
User avatar
 Dave Killoran
PowerScore Staff
  • PowerScore Staff
  • Posts: 5972
  • Joined: Mar 25, 2011
|
#67712
ray57 wrote:Hello,

I put answer choice C for this one and I am not sure why it is incorrect. In your explanation you say that C is not correct because it doesn't mention genetically modified food but neither does the correct answer, A. I chose C because it undermines the conclusion by suggesting a different cause, that lab animals COULD be fed genetically modified plants since intestinal deformities are "common at birth" thereby ruling out the author's conclusion that the genetically modified food is to blame. For answer choice A, how do we know that the rats normal diet of foods didn't include potatoes? I thought C was the surefire answer here. Can you please explain why C is wrong? Thanks for your help.
Hi Ray,

Thanks for the question! This is a great LSAT question, and so let's start with the connection to genetically modified food.

While answer choice (A) doesn't mention it explicitly, it does reference it implicitly (unlike C). When (A) says that potatoes are not normally part of a rat's diet, we accept that as true since the stimulus tells us to, and it then means that the genetically modified potatoes they were fed were abnormal to their regular diet. So, it is giving us info about genetically modified food even though it doesn't include those words. Does that make sense? In doing so, it reveals the tricky phrasing used by the stimulus: one group was "fed genetically modified potatoes" whereas the other group was "fed a normal diet of foods." Most students take that to mean that the second group was fed potatoes that were not genetically modified, but (A) shows us that rats don't even eat potatoes in labs, meaning that second group didn't even eat potatoes since they were fed a "normal diet." This shows a big difference in the diets of the two groups. So, with (A), maybe it wasn't the genetically modified food part that caused the issues, but instead the potatoes themselves.

With (C), I understand your thinking here in that maybe those deformities are just pre-existing and thus the genetically modified food didn't cause it. But, LSAC would say a few things in response to this answer:

  • 1. The argument talks specifically about developing these problems during the study, not about pre-existing issues. Thus, we aren't talking about the same issues.

    2. This answer, even if you ignored point 1 above, would apply to both groups in this study, not just the genetically modified food group. If anything, that would add some confusion since we also know the normal diet rats didn't develop these problems, but adding a point that applies to both groups does not undermine the author's explanation of a difference between the two groups.

    Essentially, we had a difference in the study and the author concludes that a difference in a part of the diet (genetically modified food) caused that difference. This similarity between the groups in this answer doesn't undermine that the difference in diets caused the issue. It's an interesting point (and one often observed in Resolve answers), and worth thinking about further.
Please let me know if that helps. Thanks!
 catatom
  • Posts: 9
  • Joined: Aug 11, 2020
|
#77978
I chose answer E. I understand that the rats who didn't develop abnormalities weren't fed anything genetically modified but it doesn't say what they were fed. But it sounds like you're saying that the answers are telling us something that we can't conclude from the passage so we accept it as true. "When (A) says that potatoes are not normally part of a rat's diet, we accept that as true since the stimulus tells us to, and it then means that the genetically modified potatoes they were fed were abnormal to their regular diet." It doesn't tell us anything about what rats are normally fed in the passage so why would we accept one answer choice as true but not any of the others?
 Luke Haqq
PowerScore Staff
  • PowerScore Staff
  • Posts: 927
  • Joined: Apr 26, 2012
|
#78718
Hi catatom!

Let's break this one down. First, you write,
When (A) says that potatoes are not normally part of a rat's diet, we accept that as true since the stimulus tells us to, and it then means that the genetically modified potatoes they were fed were abnormal to their regular diet." It doesn't tell us anything about what rats are normally fed in the passage so why would we accept one answer choice as true but not any of the others?
It isn't the stimulus but rather the question stem that tells us to treat each of the answer choices as true. The question stem is, "Which one of the following, if true, most weakens the argument?" We are therefore supposed to treat each answer choice as if it were true, given this question stem. From that vantage point, we are more specifically asked which of the answer choices would weaken the argument in the stimulus.

The stimulus argues that "Animal feed should not include genetically modified plants." Why does the author of the stimulus conclude this? The author seeks to substantiate it by citing to a study of rats that developed abnormalities after ingesting genetically modified potatoes. The author thus presumes a particular causal relationship:

cause :arrow: effect
genetically modified :arrow: abnormalities

Answer choice (A), however, calls this into question. That answer choice states, "Potatoes are not normally a part of the diet of laboratory rats." If this were true, it suggests an alternative cause, which is one key way to weaken a causal argument. Answer choice (A) suggests that it is the potatoes that might be causally responsible for the abnormalities (since they are not a normal part of their diet), rather than the genetically modification of those potatoes.

By contrast, answer choice (E) states, "The researchers conducting the study were unable to explain how the genetic modifications of the potatoes would have caused the intestinal deformities or a weakened immune system in the rats." Even if the researchers don't have 100% complete information about the mechanism of how such abnormalities occur, they still might be able to infer a causal relationship based on the studies they examine. It would have been more helpful for their conclusion, however, if those studies had compared rats eating genetically modified potatoes with rats eating non-genetically modified potatoes; this would have permitted a more accurate conclusion about whether it was the potatoes or instead the genetic modification of them that caused the abnormalities.
User avatar
 cbettaieb
  • Posts: 3
  • Joined: Sep 26, 2023
|
#103509
I answered E for this question but I was hesitant because A was also attractive, my reasoning for A was that even if potatoes are not normally a part of their diets, that does not really explain why they developed the deformities and weak immune system, unless I assume that potatoes are bad for rats or anything that is not normally part of their diet is harmful, and that is why I eliminated A because it requires additional assumptions and I thought that whenever I have to jump through assumptions to make an answer correct, it's probably not. By process of elimination, I kept E.
Please help me improve my reasoning for this type of questions, and thank you in advance!
 Rachael Wilkenfeld
PowerScore Staff
  • PowerScore Staff
  • Posts: 1419
  • Joined: Dec 15, 2011
|
#103541
Hi cbettaieb,

We are trying to weaken the argument, which means we want to make it less likely that the conclusion follows. Here, we have an experiment where some rats had genetically modified potatoes, others had their normal diet of non-genetically modified foods. The rats who ate the potatoes developed intestinal problems, and the author draws a conclusion that it was the genetic modification that was a problem. But what it if it were the potatoes? We don't know what rats' regular diet is, so if the experiment used a different type of food, and that food was also genetically modified, we don't know what variable was the relevant one.

Any time you have a causal conclusion (A caused B) suggesting an alternate cause will weaken that conclusion. By throwing out that potatoes could have been a cause, it weakened the author's conclusion that the genetic modification was a problem. Answer choice (E) doesn't have an impact on the argument at all.

Hope that helps!

Get the most out of your LSAT Prep Plus subscription.

Analyze and track your performance with our Testing and Analytics Package.